Difference between revisions of "User:Nik"

From Ferienserie MMP2
Jump to: navigation, search
Line 1: Line 1:
=== Foreword ===
+
== Foreword ==
I use \(Q\:/\:P\) instead of \(\widetilde{q}\:/\:\widetilde{p}\) because it's easier to write in Latex.  
+
I use \(Q\:/\:P\:/\:H\) for the transformed system instead of \(\widetilde{q}\:/\:\widetilde{p}\:/\:\widetilde{h}\) because it's easier to write in Latex.  
=== Problem ===
+
== Problem ==
 
Let  
 
Let  
 
\( \Phi \in C^\infty(\mathbb{R}^n) \)
 
\( \Phi \in C^\infty(\mathbb{R}^n) \)
Line 17: Line 17:
 
Show that:
 
Show that:
  
i)
 
 
\( \{Q_i(q,p), Q_j(q,p)\} = \{P_i (q,p), P_j(q,p)\} = 0 \)
 
\( \{Q_i(q,p), Q_j(q,p)\} = \{P_i (q,p), P_j(q,p)\} = 0 \)
 
ii)
 
 
\( \{Q_i(q,p), P_j(q,p)\} = \delta_{ij} \)
 
\( \{Q_i(q,p), P_j(q,p)\} = \delta_{ij} \)
  
===Attempts===
+
 
 +
==Solution==
 +
===Assumptions===
 +
From the script we use the hamilton's equations \(\dot{q_i} = \{q_i, h\} = \frac{\partial h}{\partial p_i} \text{ and } \dot{p_i} = \{p_i, h\} = -\frac{\partial h}{\partial q_i} \)
 +
 
 +
 
 +
Therefore $$ \tag{1} \dot{Q_i} = \{Q_i, h\} = \frac{\partial H}{\partial P_i} $$
 +
and $$ \tag{2} \dot{P_i} = \{P_i, h\} = -\frac{\partial H}{\partial Q_i} $$
 +
where \( H = h\circ\Phi \)
 +
 
 +
The question if we are allowed to use this is still not clear. Waiting for an answer of Raisa (02.01.15 - 10:00)
 +
 
 +
 
 +
 
 +
We also use that $$ \tag{3}  \frac{\partial h}{\partial p_l} = \sum_{k=1}^n(\frac{\partial H}{\partial Q_k}\frac{\partial Q_k}{\partial p_l} + \frac{\partial H}{\partial P_k}\frac{\partial P_k}{\partial p_l}) $$
 +
and $$ \tag{4}  \frac{\partial h}{\partial q_l} = \sum_{k=1}^n(\frac{\partial H}{\partial Q_k}\frac{\partial Q_k}{\partial q_l} + \frac{\partial H}{\partial P_k}\frac{\partial P_k}{\partial p_l}) $$
 +
which is just using the chain rule.
 +
 
 +
===Shaking variables===
 +
With these definitions it's just about shaking and changing sums:
 +
 
 +
$$ \dot{Q_i} = \{Q_i, h\} = \sum_{l=1}^n(\frac{\partial Q_i}{\partial q_l}\frac{\partial h}{\partial p_l} - \frac{\partial Q_i}{\partial p_l}\frac{\partial h}{\partial q_l}) $$
 +
With (3) and (4) it follows
 +
$$ = \sum_{l=1}^n(\frac{\partial Q_i}{\partial q_l}\sum_{j=1}^n(\frac{\partial H}{\partial Q_j}\frac{\partial Q_j}{\partial p_l} + \frac{\partial H}{\partial P_j}\frac{\partial P_j}{\partial p_l}) - \frac{\partial Q_i}{\partial p_l}\sum_{j=1}^n(\frac{\partial H}{\partial Q_j}\frac{\partial Q_j}{\partial q_l} + \frac{\partial H}{\partial P_j}\frac{\partial P_j}{\partial p_l})) $$
 +
Now we exchange the sums
 +
$$ = \sum_{j=1}^n(\frac{\partial H}{\partial Q_j}\sum_{l=1}^n(\frac{\partial Q_i}{\partial q_l}\frac{\partial Q_j}{\partial p_l} - \frac{\partial Q_i}{\partial p_l}\frac{\partial Q_j}{\partial q_l}) + \frac{\partial H}{\partial Q_j}\sum_{l=1}^n(\frac{\partial Q_i}{\partial q_l}\frac{\partial P_j}{\partial p_l} - \frac{\partial Q_i}{\partial p_l}\frac{\partial P_j}{\partial q_l})) $$
 +
And with the definition of the Poisson-brackets
 +
$$ = \sum_{j=1}^n(\frac{\partial H}{\partial Q_j}\{Q_i, Q_j\} + \frac{\partial H}{\partial Q_j}\{Q_i, P_j\}) $$
 +
To fulfill (1)
 +
$$ \implies \{Q_i, Q_j\} = 0 \text{ and } \{Q_i, P_j\} = \delta_{ij} $$
 +
 
 +
And the same for the other equation:
 +
 
 +
$$ \dot{P_i} = \{Q_i, h\} = \sum_{l=1}^n(\frac{\partial P_i}{\partial q_l}\frac{\partial h}{\partial p_l} - \frac{\partial P_i}{\partial p_l}\frac{\partial h}{\partial q_l}) $$
 +
With (3) and (4) it follows
 +
$$ = \sum_{l=1}^n(\frac{\partial P_i}{\partial q_l}\sum_{j=1}^n(\frac{\partial H}{\partial Q_j}\frac{\partial Q_j}{\partial p_l} + \frac{\partial H}{\partial P_j}\frac{\partial P_j}{\partial p_l}) - \frac{\partial P_i}{\partial p_l}\sum_{j=1}^n(\frac{\partial H}{\partial Q_j}\frac{\partial Q_j}{\partial q_l} + \frac{\partial H}{\partial P_j}\frac{\partial P_j}{\partial p_l})) $$
 +
Now we exchange the sums
 +
$$ = \sum_{j=1}^n(\frac{\partial H}{\partial Q_j}\sum_{l=1}^n(\frac{\partial P_i}{\partial q_l}\frac{\partial Q_j}{\partial p_l} - \frac{\partial P_i}{\partial p_l}\frac{\partial Q_j}{\partial q_l}) + \frac{\partial H}{\partial P_j}\sum_{l=1}^n(\frac{\partial P_i}{\partial q_l}\frac{\partial P_j}{\partial p_l} - \frac{\partial P_i}{\partial p_l}\frac{\partial P_j}{\partial q_l})) $$
 +
And with the definition of the Poisson-brackets
 +
$$ = \sum_{j=1}^n(\frac{\partial H}{\partial Q_j}\{P_i, P_j\} + \frac{\partial H}{\partial P_j}\{P_i, P_j\}) $$
 +
To fulfill (2)
 +
$$ \implies \{P_i, P_j\} = 0 \text{ and } \{P_i, Q_j\} = -\delta_{ij} =  -\{Q_i, P_j\} \:\:\:\blacksquare $$
 +
 
 +
==Attempts==
 
- Use of hamilton's equations from script page 59. I asked Raisa if we are allowed to use things from the script without proof, no answer yet (31.12.14 - 21:00)
 
- Use of hamilton's equations from script page 59. I asked Raisa if we are allowed to use things from the script without proof, no answer yet (31.12.14 - 21:00)
 
The problem there was, that you have to use a hamiltonian from the new coordinate system. May work, haven't checked that exactly.  
 
The problem there was, that you have to use a hamiltonian from the new coordinate system. May work, haven't checked that exactly.  
Line 29: Line 69:
 
- Use of simply the chain rule and insert all the things. Some solutions i saw with that said that \( \frac{\partial \Phi}{\partial p} = 0 \) but since \(\Phi \) depends on \(Q \) whicht depends on \(p \) I am not convinced with that.  
 
- Use of simply the chain rule and insert all the things. Some solutions i saw with that said that \( \frac{\partial \Phi}{\partial p} = 0 \) but since \(\Phi \) depends on \(Q \) whicht depends on \(p \) I am not convinced with that.  
  
- Use of simply the chain rule as before. At some point of this solution you should show that \( \frac{\partial Q_a}{\partial q_j}\frac{\partial}{\partial p_j}\frac{\partial}{\partial Q_j}\Phi (q, Q) =  0 \) which nobody has until now.  
+
- Use of simply the chain rule as before. At some point of this solution you should show that \( \frac{\partial Q_a}{\partial q_j}\frac{\partial}{\partial p_j}\frac{\partial}{\partial Q_j}\Phi (q, Q) =  0 \) which nobody has until now.
 
+
=== Solution for i) ===
+
=== Solution for ii) ===
+

Revision as of 09:40, 2 January 2015

Foreword

I use \(Q\:/\:P\:/\:H\) for the transformed system instead of \(\widetilde{q}\:/\:\widetilde{p}\:/\:\widetilde{h}\) because it's easier to write in Latex.

Problem

Let \( \Phi \in C^\infty(\mathbb{R}^n) \) have the property that the system \( p_i = \frac{\partial}{\partial q_i} \Phi (q, Q \) has a unique smooth solution \( Q = Q(q,p) \).

Define \( P_i(q,p) = - \frac{\partial}{\partial Q_i} \Phi (q, Q) | _{Q= Q(q,p)} \)

Let \( \{\cdot,\cdot\} \) be the Poisson bracket, such that \( \{f,g\} = \sum_{j=1}^n \frac{\partial f}{\partial q_j} \frac{\partial g}{\partial p_j} - \frac{\partial f}{\partial p_j} \frac{\partial g}{\partial q_j} \)

Show that:

\( \{Q_i(q,p), Q_j(q,p)\} = \{P_i (q,p), P_j(q,p)\} = 0 \) \( \{Q_i(q,p), P_j(q,p)\} = \delta_{ij} \)


Solution

Assumptions

From the script we use the hamilton's equations \(\dot{q_i} = \{q_i, h\} = \frac{\partial h}{\partial p_i} \text{ and } \dot{p_i} = \{p_i, h\} = -\frac{\partial h}{\partial q_i} \)


Therefore $$ \tag{1} \dot{Q_i} = \{Q_i, h\} = \frac{\partial H}{\partial P_i} $$ and $$ \tag{2} \dot{P_i} = \{P_i, h\} = -\frac{\partial H}{\partial Q_i} $$ where \( H = h\circ\Phi \)

The question if we are allowed to use this is still not clear. Waiting for an answer of Raisa (02.01.15 - 10:00)


We also use that $$ \tag{3} \frac{\partial h}{\partial p_l} = \sum_{k=1}^n(\frac{\partial H}{\partial Q_k}\frac{\partial Q_k}{\partial p_l} + \frac{\partial H}{\partial P_k}\frac{\partial P_k}{\partial p_l}) $$ and $$ \tag{4} \frac{\partial h}{\partial q_l} = \sum_{k=1}^n(\frac{\partial H}{\partial Q_k}\frac{\partial Q_k}{\partial q_l} + \frac{\partial H}{\partial P_k}\frac{\partial P_k}{\partial p_l}) $$ which is just using the chain rule.

Shaking variables

With these definitions it's just about shaking and changing sums:

$$ \dot{Q_i} = \{Q_i, h\} = \sum_{l=1}^n(\frac{\partial Q_i}{\partial q_l}\frac{\partial h}{\partial p_l} - \frac{\partial Q_i}{\partial p_l}\frac{\partial h}{\partial q_l}) $$ With (3) and (4) it follows $$ = \sum_{l=1}^n(\frac{\partial Q_i}{\partial q_l}\sum_{j=1}^n(\frac{\partial H}{\partial Q_j}\frac{\partial Q_j}{\partial p_l} + \frac{\partial H}{\partial P_j}\frac{\partial P_j}{\partial p_l}) - \frac{\partial Q_i}{\partial p_l}\sum_{j=1}^n(\frac{\partial H}{\partial Q_j}\frac{\partial Q_j}{\partial q_l} + \frac{\partial H}{\partial P_j}\frac{\partial P_j}{\partial p_l})) $$ Now we exchange the sums $$ = \sum_{j=1}^n(\frac{\partial H}{\partial Q_j}\sum_{l=1}^n(\frac{\partial Q_i}{\partial q_l}\frac{\partial Q_j}{\partial p_l} - \frac{\partial Q_i}{\partial p_l}\frac{\partial Q_j}{\partial q_l}) + \frac{\partial H}{\partial Q_j}\sum_{l=1}^n(\frac{\partial Q_i}{\partial q_l}\frac{\partial P_j}{\partial p_l} - \frac{\partial Q_i}{\partial p_l}\frac{\partial P_j}{\partial q_l})) $$ And with the definition of the Poisson-brackets $$ = \sum_{j=1}^n(\frac{\partial H}{\partial Q_j}\{Q_i, Q_j\} + \frac{\partial H}{\partial Q_j}\{Q_i, P_j\}) $$ To fulfill (1) $$ \implies \{Q_i, Q_j\} = 0 \text{ and } \{Q_i, P_j\} = \delta_{ij} $$

And the same for the other equation:

$$ \dot{P_i} = \{Q_i, h\} = \sum_{l=1}^n(\frac{\partial P_i}{\partial q_l}\frac{\partial h}{\partial p_l} - \frac{\partial P_i}{\partial p_l}\frac{\partial h}{\partial q_l}) $$ With (3) and (4) it follows $$ = \sum_{l=1}^n(\frac{\partial P_i}{\partial q_l}\sum_{j=1}^n(\frac{\partial H}{\partial Q_j}\frac{\partial Q_j}{\partial p_l} + \frac{\partial H}{\partial P_j}\frac{\partial P_j}{\partial p_l}) - \frac{\partial P_i}{\partial p_l}\sum_{j=1}^n(\frac{\partial H}{\partial Q_j}\frac{\partial Q_j}{\partial q_l} + \frac{\partial H}{\partial P_j}\frac{\partial P_j}{\partial p_l})) $$ Now we exchange the sums $$ = \sum_{j=1}^n(\frac{\partial H}{\partial Q_j}\sum_{l=1}^n(\frac{\partial P_i}{\partial q_l}\frac{\partial Q_j}{\partial p_l} - \frac{\partial P_i}{\partial p_l}\frac{\partial Q_j}{\partial q_l}) + \frac{\partial H}{\partial P_j}\sum_{l=1}^n(\frac{\partial P_i}{\partial q_l}\frac{\partial P_j}{\partial p_l} - \frac{\partial P_i}{\partial p_l}\frac{\partial P_j}{\partial q_l})) $$ And with the definition of the Poisson-brackets $$ = \sum_{j=1}^n(\frac{\partial H}{\partial Q_j}\{P_i, P_j\} + \frac{\partial H}{\partial P_j}\{P_i, P_j\}) $$ To fulfill (2) $$ \implies \{P_i, P_j\} = 0 \text{ and } \{P_i, Q_j\} = -\delta_{ij} = -\{Q_i, P_j\} \:\:\:\blacksquare $$

Attempts

- Use of hamilton's equations from script page 59. I asked Raisa if we are allowed to use things from the script without proof, no answer yet (31.12.14 - 21:00) The problem there was, that you have to use a hamiltonian from the new coordinate system. May work, haven't checked that exactly.

- Use of simply the chain rule and insert all the things. Some solutions i saw with that said that \( \frac{\partial \Phi}{\partial p} = 0 \) but since \(\Phi \) depends on \(Q \) whicht depends on \(p \) I am not convinced with that.

- Use of simply the chain rule as before. At some point of this solution you should show that \( \frac{\partial Q_a}{\partial q_j}\frac{\partial}{\partial p_j}\frac{\partial}{\partial Q_j}\Phi (q, Q) = 0 \) which nobody has until now.